# Fall 2016 (Neil-ish) ## 1 Define $$ f(x) = \sum_{n=1}^{\infty} \frac{1}{n^{x}}. $$ Show that $f$ converges to a differentiable function on $(1, \infty)$ and that $$ f'(x) =\sum_{n=1}^{\infty}\left(\frac{1}{n^{x}}\right)^{\prime}. $$ > Hint: $$ \left(\frac{1}{n^{x}}\right)^{\prime}=-\frac{1}{n^{x}} \ln n $$ ## 2 Let $f, g: [a, b] \to \RR$ be measurable with $$ \int_{a}^{b} f(x) ~d x=\int_{a}^{b} g(x) ~d x. $$ Show that either 1. $f(x) = g(x)$ almost everywhere, or 2. There exists a measurable set $E \subset [a, b]$ such that $$ \int_{E} f(x) ~d x>\int_{E} g(x) ~d x $$ ## 3 Let $f\in L^1(\RR)$. Show that $$ \lim _{x \rightarrow 0} \int_{\mathbb{R}}|f(y-x)-f(y)| d y=0 $$ ## 4 Let $(X, \mathcal M, \mu)$ be a measure space and suppose $\theset{E_n} \subset \mathcal M$ satisfies $$ \lim _{n \rightarrow \infty} \mu\left(X \backslash E_{n}\right)=0. $$ Define $$ G \definedas \theset{x\in X \suchthat x\in E_n \text{ for only finitely many } n}. $$ Show that $G \in \mathcal M$ and $\mu(G) = 0$. ## 5 Let $\phi\in L^\infty(\RR)$. Show that the following limit exists and satisfies the equality $$ \lim _{n \rightarrow \infty}\left(\int_{\mathbb{R}} \frac{|\phi(x)|^{n}}{1+x^{2}} d x\right)^{\frac{1}{n}} = \norm{\phi}_\infty. $$ ## 6 Let $f, g \in L^2(\RR)$. Show that $$ \lim _{n \rightarrow \infty} \int_{\mathbb{R}} f(x) g(x+n) d x=0 $$